University Physics with Modern Physics (14th Edition)

Published by Pearson
ISBN 10: 0321973615
ISBN 13: 978-0-32197-361-0

Chapter 29 - Electromagnetic Induction - Problems - Discussion Questions - Page 981: Q29.10

Answer

Yes, this is possible.

Work Step by Step

If the axis of rotation is parallel to the external magnetic field, there will be no flux through the loop at any time. With no change in flux, there will be no induced emf. For example, if the magnetic field were oriented vertically, the square loop should be in a vertical plane, with 2 sides horizontal and the other 2 sides vertical. There is no flux through the loop, and as the loop rotates about one of the vertical sides, there will still be no flux, and hence no emf.
Update this answer!

You can help us out by revising, improving and updating this answer.

Update this answer

After you claim an answer you’ll have 24 hours to send in a draft. An editor will review the submission and either publish your submission or provide feedback.